Calculate the slope of the curve at x=1

  • Thread starter Thread starter Kazane
  • Start date Start date
  • Tags Tags
    Curve Slope
Click For Summary
To find the slope of the curve f(x) = x^(1/3) at x = 1, the derivative f'(x) is calculated using the limit definition: f'(x) = lim (x->1) [f(x) - f(1)] / (x - 1). The expression simplifies to lim (x->1) [x^(1/3) - 1] / (x - 1). Rationalizing the numerator using the formula for the difference of cubes, a^3 - b^3 = (a - b)(a^2 + ab + b^2), is suggested to facilitate the limit calculation. Proper notation is emphasized, advising to use brackets for clarity in the limit expression. The discussion centers on correctly applying these mathematical principles to compute the derivative.
Kazane
Messages
5
Reaction score
0

Homework Statement



f(x)=x1/3
use the definition of f'(a) to calculate the slope of the curve at x=1
(Hint: by rationalizing the numerator. useful formula a3 -b3=(a-b)(a2+ab+b2)

The Attempt at a Solution


f'(x)=lim (x->1) f(x)-f(1)/x-1

=lim (x->1) x1/3-11/3/ x-1

How can I do x1/3-11/3 to a3 -b3=(a-b)(a2+ab+b2)?

I'm confused so please help!
 
Physics news on Phys.org


Kazane said:

Homework Statement



f(x)=x1/3
use the definition of f'(a) to calculate the slope of the curve at x=1
(Hint: by rationalizing the numerator. useful formula a3 -b3=(a-b)(a2+ab+b2)



The Attempt at a Solution


f'(x)=lim (x->1) f(x)-f(1)/x-1

=lim (x->1) x1/3-11/3/ x-1

How can I do x1/3-11/3 to a3 -b3=(a-b)(a2+ab+b2)?

I'm confused so please help!

You should never write f(x) - f(1)/x-1, which means f(x) - [f(1)/x] - 1 when evaluated according to standard, universally accepted rules. You should use brackets and write [f(x) - f(1)]/(x-1).

RGV
 


Kazane said:

Homework Statement



f(x)=x1/3
use the definition of f'(a) to calculate the slope of the curve at x=1
(Hint: by rationalizing the numerator. useful formula a3 -b3=(a-b)(a2+ab+b2)



The Attempt at a Solution


f'(x)=lim (x->1) f(x)-f(1)/x-1

=lim (x->1) x1/3-11/3/ x-1

How can I do x1/3-11/3 to a3 -b3=(a-b)(a2+ab+b2)?

I'm confused so please help!
Let a= x^{1/3}, b= y^{1/3}
 
Question: A clock's minute hand has length 4 and its hour hand has length 3. What is the distance between the tips at the moment when it is increasing most rapidly?(Putnam Exam Question) Answer: Making assumption that both the hands moves at constant angular velocities, the answer is ## \sqrt{7} .## But don't you think this assumption is somewhat doubtful and wrong?

Similar threads

Replies
2
Views
1K
  • · Replies 12 ·
Replies
12
Views
2K
  • · Replies 7 ·
Replies
7
Views
2K
  • · Replies 9 ·
Replies
9
Views
2K
  • · Replies 1 ·
Replies
1
Views
1K
  • · Replies 5 ·
Replies
5
Views
2K
  • · Replies 1 ·
Replies
1
Views
1K
  • · Replies 14 ·
Replies
14
Views
2K
Replies
4
Views
1K
Replies
3
Views
1K